Integrating a solid using cartesian, cylindrical and spherical coordinates












0












$begingroup$


problem




The region $W$ is the cone shown below (see image).



The angle at the vertex is $π/3$, and the top is flat and at a height of $7sqrt{3}$.
Write the limits of integration for $int_W dV$ in the following coordinates (do not reduce the domain of integration by taking advantage of symmetry):




Progress



As seen in the image, the ones highlighted in red are those I cannot figure out. I'm not sure if I have some fundamental misconceptions that allows me to derive some of the boundaries but not others, but if someone can kindly explain how to find those values in red, I would be grateful. Thanks in advance.










share|cite|improve this question











$endgroup$








  • 1




    $begingroup$
    Is this your homework or something? There is ""(1 pt)"" attached to the question.
    $endgroup$
    – Aaron Maroja
    Nov 29 '14 at 0:12


















0












$begingroup$


problem




The region $W$ is the cone shown below (see image).



The angle at the vertex is $π/3$, and the top is flat and at a height of $7sqrt{3}$.
Write the limits of integration for $int_W dV$ in the following coordinates (do not reduce the domain of integration by taking advantage of symmetry):




Progress



As seen in the image, the ones highlighted in red are those I cannot figure out. I'm not sure if I have some fundamental misconceptions that allows me to derive some of the boundaries but not others, but if someone can kindly explain how to find those values in red, I would be grateful. Thanks in advance.










share|cite|improve this question











$endgroup$








  • 1




    $begingroup$
    Is this your homework or something? There is ""(1 pt)"" attached to the question.
    $endgroup$
    – Aaron Maroja
    Nov 29 '14 at 0:12
















0












0








0





$begingroup$


problem




The region $W$ is the cone shown below (see image).



The angle at the vertex is $π/3$, and the top is flat and at a height of $7sqrt{3}$.
Write the limits of integration for $int_W dV$ in the following coordinates (do not reduce the domain of integration by taking advantage of symmetry):




Progress



As seen in the image, the ones highlighted in red are those I cannot figure out. I'm not sure if I have some fundamental misconceptions that allows me to derive some of the boundaries but not others, but if someone can kindly explain how to find those values in red, I would be grateful. Thanks in advance.










share|cite|improve this question











$endgroup$




problem




The region $W$ is the cone shown below (see image).



The angle at the vertex is $π/3$, and the top is flat and at a height of $7sqrt{3}$.
Write the limits of integration for $int_W dV$ in the following coordinates (do not reduce the domain of integration by taking advantage of symmetry):




Progress



As seen in the image, the ones highlighted in red are those I cannot figure out. I'm not sure if I have some fundamental misconceptions that allows me to derive some of the boundaries but not others, but if someone can kindly explain how to find those values in red, I would be grateful. Thanks in advance.







integration multivariable-calculus volume






share|cite|improve this question















share|cite|improve this question













share|cite|improve this question




share|cite|improve this question








edited Dec 12 '14 at 19:32







user147263

















asked Nov 28 '14 at 23:28









eloggingelogging

3829




3829








  • 1




    $begingroup$
    Is this your homework or something? There is ""(1 pt)"" attached to the question.
    $endgroup$
    – Aaron Maroja
    Nov 29 '14 at 0:12
















  • 1




    $begingroup$
    Is this your homework or something? There is ""(1 pt)"" attached to the question.
    $endgroup$
    – Aaron Maroja
    Nov 29 '14 at 0:12










1




1




$begingroup$
Is this your homework or something? There is ""(1 pt)"" attached to the question.
$endgroup$
– Aaron Maroja
Nov 29 '14 at 0:12






$begingroup$
Is this your homework or something? There is ""(1 pt)"" attached to the question.
$endgroup$
– Aaron Maroja
Nov 29 '14 at 0:12












1 Answer
1






active

oldest

votes


















0












$begingroup$

The cross-sectional radius of the cone increases linearly with $z$, so the lower limit of $z$ also increases linearly. So $1/7$th of the way up, the lower $z$ limit needs to be $sqrt{3}$. From your bounds on $x$ and $y$, $sqrt{x^2 + y^2} = sqrt{2}$ at $z=1$, so the limits of $z$ are $left(sqrt{3/2}sqrt{x^2+y^2}right)$ to $7sqrt{3}.$



For the cylindrical coordinates, it's much the same. Replace $sqrt{x^2+y^2}$ with $r$. (You should have $r$ as the integrand instead of $rho$.)



For the spherical coordinates, the $rho$ dependence is with $phi$ only. At $phi=0$, $rho = 7sqrt{3}$, while at $phi=pi/6$, $rho=7sqrt{3}/cos(pi/6).$ So your upper limit is $7sqrt{3}/cosphi$.



The integrand is determined from the spherical volume element, which is



$$dV = rho^2 sin theta; drho; dtheta; dphi.$$






share|cite|improve this answer











$endgroup$













    Your Answer





    StackExchange.ifUsing("editor", function () {
    return StackExchange.using("mathjaxEditing", function () {
    StackExchange.MarkdownEditor.creationCallbacks.add(function (editor, postfix) {
    StackExchange.mathjaxEditing.prepareWmdForMathJax(editor, postfix, [["$", "$"], ["\\(","\\)"]]);
    });
    });
    }, "mathjax-editing");

    StackExchange.ready(function() {
    var channelOptions = {
    tags: "".split(" "),
    id: "69"
    };
    initTagRenderer("".split(" "), "".split(" "), channelOptions);

    StackExchange.using("externalEditor", function() {
    // Have to fire editor after snippets, if snippets enabled
    if (StackExchange.settings.snippets.snippetsEnabled) {
    StackExchange.using("snippets", function() {
    createEditor();
    });
    }
    else {
    createEditor();
    }
    });

    function createEditor() {
    StackExchange.prepareEditor({
    heartbeatType: 'answer',
    autoActivateHeartbeat: false,
    convertImagesToLinks: true,
    noModals: true,
    showLowRepImageUploadWarning: true,
    reputationToPostImages: 10,
    bindNavPrevention: true,
    postfix: "",
    imageUploader: {
    brandingHtml: "Powered by u003ca class="icon-imgur-white" href="https://imgur.com/"u003eu003c/au003e",
    contentPolicyHtml: "User contributions licensed under u003ca href="https://creativecommons.org/licenses/by-sa/3.0/"u003ecc by-sa 3.0 with attribution requiredu003c/au003e u003ca href="https://stackoverflow.com/legal/content-policy"u003e(content policy)u003c/au003e",
    allowUrls: true
    },
    noCode: true, onDemand: true,
    discardSelector: ".discard-answer"
    ,immediatelyShowMarkdownHelp:true
    });


    }
    });














    draft saved

    draft discarded


















    StackExchange.ready(
    function () {
    StackExchange.openid.initPostLogin('.new-post-login', 'https%3a%2f%2fmath.stackexchange.com%2fquestions%2f1042976%2fintegrating-a-solid-using-cartesian-cylindrical-and-spherical-coordinates%23new-answer', 'question_page');
    }
    );

    Post as a guest















    Required, but never shown

























    1 Answer
    1






    active

    oldest

    votes








    1 Answer
    1






    active

    oldest

    votes









    active

    oldest

    votes






    active

    oldest

    votes









    0












    $begingroup$

    The cross-sectional radius of the cone increases linearly with $z$, so the lower limit of $z$ also increases linearly. So $1/7$th of the way up, the lower $z$ limit needs to be $sqrt{3}$. From your bounds on $x$ and $y$, $sqrt{x^2 + y^2} = sqrt{2}$ at $z=1$, so the limits of $z$ are $left(sqrt{3/2}sqrt{x^2+y^2}right)$ to $7sqrt{3}.$



    For the cylindrical coordinates, it's much the same. Replace $sqrt{x^2+y^2}$ with $r$. (You should have $r$ as the integrand instead of $rho$.)



    For the spherical coordinates, the $rho$ dependence is with $phi$ only. At $phi=0$, $rho = 7sqrt{3}$, while at $phi=pi/6$, $rho=7sqrt{3}/cos(pi/6).$ So your upper limit is $7sqrt{3}/cosphi$.



    The integrand is determined from the spherical volume element, which is



    $$dV = rho^2 sin theta; drho; dtheta; dphi.$$






    share|cite|improve this answer











    $endgroup$


















      0












      $begingroup$

      The cross-sectional radius of the cone increases linearly with $z$, so the lower limit of $z$ also increases linearly. So $1/7$th of the way up, the lower $z$ limit needs to be $sqrt{3}$. From your bounds on $x$ and $y$, $sqrt{x^2 + y^2} = sqrt{2}$ at $z=1$, so the limits of $z$ are $left(sqrt{3/2}sqrt{x^2+y^2}right)$ to $7sqrt{3}.$



      For the cylindrical coordinates, it's much the same. Replace $sqrt{x^2+y^2}$ with $r$. (You should have $r$ as the integrand instead of $rho$.)



      For the spherical coordinates, the $rho$ dependence is with $phi$ only. At $phi=0$, $rho = 7sqrt{3}$, while at $phi=pi/6$, $rho=7sqrt{3}/cos(pi/6).$ So your upper limit is $7sqrt{3}/cosphi$.



      The integrand is determined from the spherical volume element, which is



      $$dV = rho^2 sin theta; drho; dtheta; dphi.$$






      share|cite|improve this answer











      $endgroup$
















        0












        0








        0





        $begingroup$

        The cross-sectional radius of the cone increases linearly with $z$, so the lower limit of $z$ also increases linearly. So $1/7$th of the way up, the lower $z$ limit needs to be $sqrt{3}$. From your bounds on $x$ and $y$, $sqrt{x^2 + y^2} = sqrt{2}$ at $z=1$, so the limits of $z$ are $left(sqrt{3/2}sqrt{x^2+y^2}right)$ to $7sqrt{3}.$



        For the cylindrical coordinates, it's much the same. Replace $sqrt{x^2+y^2}$ with $r$. (You should have $r$ as the integrand instead of $rho$.)



        For the spherical coordinates, the $rho$ dependence is with $phi$ only. At $phi=0$, $rho = 7sqrt{3}$, while at $phi=pi/6$, $rho=7sqrt{3}/cos(pi/6).$ So your upper limit is $7sqrt{3}/cosphi$.



        The integrand is determined from the spherical volume element, which is



        $$dV = rho^2 sin theta; drho; dtheta; dphi.$$






        share|cite|improve this answer











        $endgroup$



        The cross-sectional radius of the cone increases linearly with $z$, so the lower limit of $z$ also increases linearly. So $1/7$th of the way up, the lower $z$ limit needs to be $sqrt{3}$. From your bounds on $x$ and $y$, $sqrt{x^2 + y^2} = sqrt{2}$ at $z=1$, so the limits of $z$ are $left(sqrt{3/2}sqrt{x^2+y^2}right)$ to $7sqrt{3}.$



        For the cylindrical coordinates, it's much the same. Replace $sqrt{x^2+y^2}$ with $r$. (You should have $r$ as the integrand instead of $rho$.)



        For the spherical coordinates, the $rho$ dependence is with $phi$ only. At $phi=0$, $rho = 7sqrt{3}$, while at $phi=pi/6$, $rho=7sqrt{3}/cos(pi/6).$ So your upper limit is $7sqrt{3}/cosphi$.



        The integrand is determined from the spherical volume element, which is



        $$dV = rho^2 sin theta; drho; dtheta; dphi.$$







        share|cite|improve this answer














        share|cite|improve this answer



        share|cite|improve this answer








        edited Nov 29 '14 at 0:38

























        answered Nov 29 '14 at 0:32









        JohnJohn

        22.6k32450




        22.6k32450






























            draft saved

            draft discarded




















































            Thanks for contributing an answer to Mathematics Stack Exchange!


            • Please be sure to answer the question. Provide details and share your research!

            But avoid



            • Asking for help, clarification, or responding to other answers.

            • Making statements based on opinion; back them up with references or personal experience.


            Use MathJax to format equations. MathJax reference.


            To learn more, see our tips on writing great answers.




            draft saved


            draft discarded














            StackExchange.ready(
            function () {
            StackExchange.openid.initPostLogin('.new-post-login', 'https%3a%2f%2fmath.stackexchange.com%2fquestions%2f1042976%2fintegrating-a-solid-using-cartesian-cylindrical-and-spherical-coordinates%23new-answer', 'question_page');
            }
            );

            Post as a guest















            Required, but never shown





















































            Required, but never shown














            Required, but never shown












            Required, but never shown







            Required, but never shown

































            Required, but never shown














            Required, but never shown












            Required, but never shown







            Required, but never shown







            Popular posts from this blog

            To store a contact into the json file from server.js file using a class in NodeJS

            Redirect URL with Chrome Remote Debugging Android Devices

            Dieringhausen